CỰC TRỊ TẬP HỢP 2 MỘT SỐ PHƯƠNG PHÁP GIẢI TOÁN CỰC TRỊ TẬP HỢP 0 ≤x−SX<1.

Một phần của tài liệu Bộ đề thi năng khiếu môn toán lớp 10 trường chuyên năm 2022 (Trang 90 - 93)

- Nế uT thuộc tập K, thì A lại có thể chọn ngườ i1 bắt cặp với T, quá trình cứ lặp lại như vậy rồi sẽ đến lúc B phải chọn được một người “T” nào đó không thuộc tập

CỰC TRỊ TẬP HỢP 2 MỘT SỐ PHƯƠNG PHÁP GIẢI TOÁN CỰC TRỊ TẬP HỢP 0 ≤x−SX<1.

2. MỘT SỐ PHƯƠNG PHÁP GIẢI TOÁN CỰC TRỊ TẬP HỢP

CỰC TRỊ TẬP HỢP 2 MỘT SỐ PHƯƠNG PHÁP GIẢI TOÁN CỰC TRỊ TẬP HỢP 0 ≤x−SX<1.

0≤xSX <1. + Xét vớix>1+a+a2+···+an= a n+1−1 a−1 , khi đó vì an+1−1 a−1 = a n+1 −1 3 2−1 =2 an+1−1=an+1+ (an+1−2)>an+1+a2−2=an+1+1 4>a n+1 nên 0<(xan+1)≤1+a+a2+···+an.

Theo giả thiết quy nạp, tồn tại tập conXQn thỏa mãn

0≤(xan+1)−SX <1 =⇒ 0≤xSX′ <1

với

X′=X[{an+1} ⊂Qn+1. Bài toán được chứng minh.

Ví dụ 2.5.4(Romania, grade 9, final round). Cho p,qlà hai số nguyên dương, p≥2,q≥2. Một tập

hữu hạnX gọi là có tính chất (S)nếu: với bất kỳ cách chọn ptập con BiX,i =1,2, . . . ,p, không nhất thiết phân biệt, mỗi tập cóqphần tử, thì luôn tồn tại tập conYXpphần tử, sao cho

|YBi| ≤2,∀i=1,2, . . . ,p. Chứng minh rằng

1. Mọi tậpXpqqphần tử đều không có tính chất (S). 2. Mọi tập hợpXpqq+1phần tử đều có tính chất (S).

Chứng minh. 1. NếuXpqq= (p−1)qphần tử, khi đó ta chọn p−1tập, mỗi tập có qphần

tử (là phân hoạch củaX) như sau

B1={1,2, . . . ,q}, B2 ={q+1,q+2, . . . ,2q}, . . . , Bp−1={(p−2)q+1, . . . ,(p−1)q}

và tập Bp là một tập tùy ý cóqphần tử. Khi đó, theo nguyên lý Dirichlet, thì với mọi tậpYp

phần tử trong X, sẽ tồn tại ít nhất một tậpBi (i∈ {1,2, . . . ,p−1}) sao cho giao của nó vớiY

≥2phần tử. Vậy tậpX không có tính chất(S).

2. Bây giờ với mỗiicho trước, nhận xét tập

[

j6=i

Bi

có ≤(p−1)qphần tử. Trong khi đó tập Xpqq+1= (p−1)q+1phần tử. Do đó ta tìm đượcít nhất một phần tử của tậpX mà nó không nằm trong mọi tậpBj, với mọi j6=i.

• Với i = 1, sử dụng lập luận trên, ta tìm được phần tử x1 mà x1 6∈Bj,∀j =2, . . . ,p. Nếu

x1∈B1, ta tiếp tục qua bước 2, nếux16∈B1, ta xây dựng tậpB1 mới bằng cách bỏ ra khỏi

B1một phần tửy1, và thêm vào tậpB1 phần tửx1.Dĩ nhiên phần tửy1vẫn là phần tử trong

X.

CỰC TRỊ TẬP HỢP 2 MỘT SỐ PHƯƠNG PHÁP GIẢI TOÁN CỰC TRỊ TẬP HỢP

• Tiếp tục vớii=2, lưu ý tậpB1 sử dụng bây giờ là tậpB1 "mới". Lại sử dụng lập luận trên, ta tìm được phần tửx2 màx2 không nằm trong bất cứ tậpB1,B3,B4, . . . ,Bn. Khi đóx26=x1

(vìx1∈B1,x26∈B1). Nếux2∈B2 ta tiếp tục quy trình, nếux26∈B2, ta xây dựng tậpB2 mới bằng cách bỏ ra khỏiB2 một phần tửy2, và thêm vào tậpB2 phần tửx2.

3. Cứ tiếp tục quy trình này đến bước p, ta sẽ tìm được tậpY ={x1,x2, . . . ,xp}. TậpY giao với mỗi tập Bi "mới" chính xác một phần tử, đó làxi. Bây giờ thay ngược trở lại xi bởiyi ta quay trở lại tậpBi"cũ". Nếuyi6∈Y thìYBi= /0, còn nếuyiY thìYBi={yi}.

Ví dụ 2.5.5(China TST 2015). Cho X là một tập khác rỗng và A1,A2, . . . ,Anntập con củaX sao

cho

1. |Ai| ≤3,∀i=1,2, . . . ,n;

2. Bất kỳ một phần tử nào củaX cũng nằm trong ít nhất 4 tập trong sốA1,A2, . . . ,An.

Chứng minh rằng có thể chọn3n

7

tập hợp trong số các tậpA1,A2, . . . ,An mà hợp của chúng bằngX.

Chứng minh. Kết luận bài toán yêu cầu chọn được một số tập hợp, hợp lại bằngX, do đó ta sẽ

1. Chọn tập đầu tiên có 3 phần tử, giả sử A1,|A1|=3. Sau đó, ta chọn tiếp tậpA2 |A2|=3 A2∩A1= /0. Sau khi chọn được tậpA2, ta chọn tiếp tậpA3|A3|=3vàA3∩A1= /0,A3∩A2= /0, cứ tiếp tục như vậy đến khi không thể chọn thêm được tập nào nữa vào trong hệ. Trong tất cả cách cách lựa chọn các tập hợp trong A1,A2, . . . ,An, ta chọn ra hệ tập hợp S3 cực đại, giả sử

S3 ={A1,A2, . . . ,Ai}(in) (tức là họ S3 chứa nhiều tập hợp nhất có thể có) mà |At|=3,∀t = 1,2, . . . ,iArAs= /0,∀1≤r<si(điều này có nghĩa, mỗi lần bổ sung một tập hợp vàoS3

thì tậpX3 có số lượng phần tử tăng lên 3).

X X\X3 X3 b b b b b b b b b b b b b b b b

ĐặtX3=SArS3Ar. Do tính tối đại của tậpS3, nên với bất kỳ tập hợpAj (j>i) thì

|Aj∩(X\X3)| ≤2

vì nếu không thì ta tiếp tục bổ sungAj vào tậpS3, mâu thuẫn với tính tối đại củaS3. Và khi đó

|X3|=3i.

CỰC TRỊ TẬP HỢP 2 MỘT SỐ PHƯƠNG PHÁP GIẢI TOÁN CỰC TRỊ TẬP HỢP

2. Bây giờ ta tiếp tục chọn họS2 cực đạichứa các tậpAjcòn lại trong sốAi+1, . . . ,An, sao chomỗi lần thêm một tập hợp vào họ S2, thì số lượng phần tử trong hợp của chúng tăng lên 2. Không mất tính tổng quát, giả sử S2={Ai+1,Ai+2, . . . ,Aj} X X\X3 X3 X2 b b b b b b b b b b b b b b b b b b b b b b b b b Đặt X2 = [ ArS2 Ar∩(X\X3)

thì theo cách xác định củaS2 ta có|X2|=2 j và theo tính tối đại của tậpS2 thì

|At∩(X\(X2∪X3))| ≤1,∀t = j+1, . . . ,n.

3. Bây giờ ta tiếp tục chọn họ S1 chứa các tập As còn lại trong số Aj+1, . . . ,An, sao cho mỗi lần thêm một tập hợp vào họ S1, thì số lượng phần tử trong hợp của chúng tăng lên 1dĩ nhiên các tập hợp trong họ S1 chứa hết tất cả các phần tử của X\(X3∪X2) =X1. Không mất tính tổng quát, giả sử

S1={Aj+1,Ai+2, . . . ,Ak}. 4. Khi đó |X|=|X1|+|X2|+|X3|=3i+2 j+k,X =X1∪X2∪X3 và

|S3|+|S2|+|S1|=i+ j+k=m. Ta cần chứng minhm3n7 .

• Vì mỗi phần tử trongX1nằm trong ít nhất 4 tập hợp, nhưng do|ArX1| ≤1,∀r= j+1, . . . ,n

nên

ni+j+4k. (1)

Mỗi phần tử trong X1∪X2 xuất hiện ít nhất trong 4 tập hợp, như do |Ar| ∩(X1∪X2)≤

2,∀r=i+1, . . . ,nnên

ni+4(2 j+k)

2 =i+4 j+2k. (2)

Mỗi phần tử trong X xuất hiện ít nhất trong 4 tập hợp, và|ArX| ≤3,∀r=1,2, . . . ,n, do đó

n≥ 4(3i+32 j+k). (3)

CỰC TRỊ TẬP HỢP 2 MỘT SỐ PHƯƠNG PHÁP GIẢI TOÁN CỰC TRỊ TẬP HỢP• Lấy20×(1) +12×(2) +27×(3)ta có

Một phần của tài liệu Bộ đề thi năng khiếu môn toán lớp 10 trường chuyên năm 2022 (Trang 90 - 93)